5
$\begingroup$

At the risk of this question having an easy answer, I am asking the following:

Suppose we have a measurable cardinal $\kappa$ and a (possibly regular) cardinal $\lambda>\kappa$. Is there any condition, like the existence of an appropriate ultrafilter, that implies the existence of an elementary embedding $j:V\to M$ with critical point $\kappa$ so that $j(\kappa)=\lambda$?

An example is when $\kappa$ is huge: if there is a normal $\kappa$-complete ultrafilter $U$ on $P(\lambda)$ with the property that $\{X\subseteq P(\lambda)\mid ot(X)=\kappa\}\in U$, then the ultrapower embedding $j:V\to M\simeq Ult(V,U)$ satisfies $j(\kappa)=\lambda$ (and $~{}^{j(\kappa)}M\subseteq M$)..

However, I am interested in the case where $\kappa$ is only measurable, maybe not even strong.

I am assuming Choice and whenever I write $j:V\to M$ I mean that $M\subseteq V$ is a transitive model of ZFC.

$\endgroup$

1 Answer 1

10
$\begingroup$

Here is a necessary and sufficient criterion: $\lambda>2^\kappa$. This is easily seen to be necessary, since if $j:V\to M$ has critical point $\kappa$, then the power set $P(\kappa)$ is contained in $M$, and from this it follows that $j(\kappa)>2^\kappa$ since $j(\kappa)$ is inaccessible in $M$. Conversely, we can hit every cardinal above $2^\kappa$, by the following.

Theorem. If $\kappa$ is a measurable cardinal, then every cardinal $\lambda>2^\kappa$ is the image $\lambda=j(\kappa)$ of some elementary embedding $j:V\to M$ with critical point $\kappa$.

Proof. Let $\mu$ be a normal measure on $\kappa$ and consider the class of images $j_\alpha(\kappa)$, where $j_\alpha$ is the $\alpha$-iterated ultrapower by $\mu$. These are the ordinals that form the critical sequence. This class of ordinals is closed and unbounded. It is clearly unbounded, since by iterating further, we can push $j(\kappa)$ as high as desired. It is closed, since the iterations are defined to take the direct limit at limit stages, and this makes the critical sequence continuous.

Finally, I claim that every cardinal above $2^\kappa$ is on the critical sequence. To see this, it suffices to argue that at each stage, we don't jump over the next cardinal. If $j_\alpha:V\to M_\alpha$ is the $\alpha^{th}$ iterate, then the extender representation shows that every element of $M$ has the form $j(f)(s)$, where $f:\kappa\to V$ and $s$ is a finite sequence from the critical sequence below $\kappa_\alpha$, which are ordinals below $j_\alpha(\kappa)$. If we go one more step, to $j_{\alpha+1}:V\to M_{\alpha+1}$, then we only need to add one more generator or seed, namely $\kappa_\alpha$ itself, and so $|j_{\alpha+1}(\kappa)|^V\leq |\kappa^\kappa|\cdot|(\alpha+1)^{<\omega}|$, which has the same size as $j_\alpha(\kappa)$. So at successor stages, we don't get to the next cardinal, and so we reach all the cardinals at limit stages. So we'll get every cardinal above $2^\kappa$. QED

Lastly, note that although you had asked about cardinals $\lambda$ that are the images of $\kappa$ under an embedding, nevertheless such images are not always cardinals. For example, if $\mu$ is a measure on $\kappa$ with ultrapower $j:V\to M$, then $j(\kappa)$ is never a cardinal, since it is strictly between $2^\kappa$ and $(2^\kappa)^+$. So one might want a criterion for recognizing when an ordinal $\lambda$ is the image of $\kappa$ under an embedding, and I think that is a much subtler question.

$\endgroup$
15
  • $\begingroup$ Thanks Joel, very nice answer! I had not realised that when we iterate a normal measure, the critical sequence is a club. Although this is a totally satisfying answer, I am still hoping that the existence of a suitable ultrafilter (on $P(\lambda)$ or on $[\lambda]^\kappa$) could also give the same result, like in the case of hugeness. Also, I agree with your final remark, it would be interesting to have absolute control on where the critical point is mapped. $\endgroup$ Feb 1, 2017 at 15:23
  • $\begingroup$ That is a subtle point, @Stamatis, that the embedding might not be the result of an ultrapower by a normal (or any) measure. $\endgroup$
    – Asaf Karagila
    Feb 1, 2017 at 19:04
  • $\begingroup$ Actually, it's a theorem @StamatisDimopoulos that the existence of a $\kappa$ such that ultrapowers have $j(\kappa)$ arbitrarily large is equiconsistent with the existence of a strong cardinal. Such a $\kappa$ is called a "tall" cardinal. $\endgroup$ Oct 5, 2018 at 0:03
  • $\begingroup$ @KeithMillar Retrospectively, I agree with you, we need at least a strong cardinal. However, isn't there a distinction between what Hamkins calls tall and strongly tall cardinals? In the former case we have merely extender embeddings, while in the latter we have ultrapower embeddings. I had the impression that we still don't know the exact consistency strength of strongly tall cardinals, do you know if that has changed? $\endgroup$ Oct 5, 2018 at 10:16
  • $\begingroup$ @StamatisDimopoulos I believe this is a debate of semantics; Jech's set theory (the textbook I used mainly) considers ultrapowers to include extender embeddings. However, I also think that when I wrote this comment I did in fact mean ultrapowers on measures, and I'm not sure what I was thinking. $\endgroup$ Oct 7, 2018 at 4:16

Your Answer

By clicking “Post Your Answer”, you agree to our terms of service and acknowledge you have read our privacy policy.

Not the answer you're looking for? Browse other questions tagged or ask your own question.